0 Daumen
779 Aufrufe

Aufgabe:

 10 Jungs und 10 Mädels stehen in zufälliger Aufreihung in einer Schlange. Bestimme die zu erwartende Anzahl an Jungen, die neben einem Mädchen stehen.


Problem/Ansatz:

… Man könnte hier zunächst einmal die Anzahl der Möglichkeiten bestimmen. 10!

Und dann folgendes Schema abarbeiten:

a = Junge, b = Mädchen

aaaaabbbbb

Hier sind durchschnittlich gesehen 0,2 Jungen neben einem Mädchen. Und diese Möglichkeit gibt es 5!5! * 2, da die Mädchenreihe auch vorne sein kann.

Also hat man schonmal (0.2*5!5!*2 + ...)  / 10!

Bei so einer großen Anzahl an Personen, scheint diese Methode aber nicht sehr effizient zu sein. Gibt es noch eine andere Möglichkeit, wie man hier vorgehen könnte?

Avatar von

Wenn 20 Leute in einer Schlange hintereinander stehen, ist die Wahrscheinlichkeit, dass jemand neben einem steht gleich Null. Oder habe ich etwas falsch verstanden?

Wenn 20 Leute in einer Schlange hintereinander stehen

Richtig und Aufgabe b lautet wie sieht es aus, wenn es statt einer Schlange eine Reihe wäre?

blob.png

Hier stehen 9 Mädchen nicht neben einem Jungen und ein Mädchen steht neben einem Jungen. Durschnittlich gesehen steht hier also genau 0,1 Junge neben einem Mädchen.

Welches Programm kann den gesuchten Erwartungswert berechnen?

Welches Programm kann den gesuchten Erwartungswert berechnen?

Ein selbstgeschriebenes?

Du hast ja schon gemerkt das es exakt wohl sehr viel Mühe macht das auszurechnen. Ja. länger do eine Reihe ist desto besser ließe sich die Wahrscheinlichkeit wohl auch mitteln Näherungen berechnen. Man könnte das also mal für kleinere Reihen exakt und näherungsweise machen. Manchmal stolpert man dann auch über etwas, wie man es vielleicht geschickt vereinfachen kann.

Hallo

1, ich denke die J und die M sind ununterscheidbar, deshalb denk lieber an rote und weisse Kugeln .dann gibt es nur 2 Möglichkeiten für nur 1 Nachbarmädchen. wieviel gibt es für 2,... wieviel für 5

Gruß lul

Macht man mal eine Allgemeine Betrachtung. Ich gehe davon aus das immer gleichviele Jungen wie Mädchen vorhanden sind

JM, MJ → Erwartungsgemäß sitzt 1 Junge neben einem Mädchen

JJMM, JMJM, JMMJ, MJJM, MJMJ, MMJJ → Erwartungsgemäß sitzt 1 Junge neben einem Mädchen, wenn ich mich nicht verrechnet habe.

Könnt man vielleicht nachweisen, dass der Erwartungswert immer 1 ist? Mir kommt da irgendwie vollständige Induktion in den Sinn. Aber irgendwie scheint der Induktionsschritt nicht ganz so trivial zu sein.

Könnt man vielleicht nachweisen, dass der Erwartungswert immer 1 ist?

Warum sollte das so sein? Das ist ja nicht mal in deinem Beispiel der Fall.

Deine 6 Reihenfolgen sind gleich wahrscheinlich (also 1/6). Die Anzahl der Jungen, die neben sich ein Mädchen sitzen haben, ist in deinen Fällen:

1, 2, 2, 2, 2,1.

Der Erwartungswert für die Anzahl der Jungen, die neben einem Mädchen sitzen, ist also 10/6.

Könnt man vielleicht nachweisen, dass der Erwartungswert immer 1 ist?

Für 3 Jungen und 3 Mädchen komme ich auch auf einen Erwartungswert von 1.

blob.png

$$\mu= \left(2\cdot\frac{1}{3} +4\cdot\frac{2}{3} + 8\cdot\frac{3}{3} + 4\cdot\frac{4}{3} + 2\cdot\frac{5}{3}\right) : 20 = \underline{\underline{1}}$$  
Wie könnte man nachweisen, dass die Ergebnismenge (hier: 2-4-8-4-2) immer symmetrisch ist?

@abakus

Offensichtlich hast du die Aufgabe anders aufgefasst als ich. Kein Problem. Vielleicht erklärt Hikoba es etwas genauer wie er die Aufgabe gemeint hat.

@hikoba

warum teilst du durch 20 du hast doch nur 5 Ereignisse von 1 bis 5? oder wie kommst du auf die Drittel? wenn du durch 20  Teilst brauchst du doch die Ganzen Ereignisse also 2*1+...?

lul

warum teilst du durch 20 du hast doch nur 5 Ereignisse von 1 bis 5?

Ich denke das sind die 20 Reihen die du in der Skizze siehst.

Ja aber dann woher die Drittel? Der Erwartungswert 1 muss ja falsch sein ausser bei 1J 1M

lul

Ja aber dann woher die Drittel? Der Erwartungswert 1 muss ja falsch sein ausser bei 1J 1M

Kann es sein das du es wie abakus interpretierst und ich es wie Hikoba interpretiere?

Es ist gefährlich wenn man von unterschiedlichen Dingen spricht aber meint es sei das gleiche. Dann ist es schwer auf einen Nenner zu kommen.

Ich betrachte nochmals 2 Jungen und 2 Mädchen. Jedes Mädchen zählt nun die Jungen die direkt neben Ihr stehen

JJMM - 1 0
JMJM - 2 1
JMMJ - 1 1
MJJM - 1 1
MJMJ - 1 2
MMJJ - 0 1

Also alle Mädchen zusammen zählen 12 Jungen direkt neben sich. Da es insgesamt 12 Mädchen waren zählt jeden Mädchen im Mittel genau einen Jungen neben sich.

Verstehe ich nicht, wieso insgesamt 12 Mädchen?

ich zähle 6 Aufstellungen in 2 Aufstellungen gibt es eine Paar und , in 4 Aufstellungen 2 Paare

also (2*1+4*2)/6

Man sieht doch direkt dass 2 J neben einem M stehen häufiger ist als 1 J neben 1 M, deshalb ist direkt klar dass der EW NICHT 1 sein kann .

lul

In jeder Aufstellung gibt es 2 Mädchen und 2 Jungen.

Es gibt 6 Aufstellungen und daher 6 * 2 Mädchen = 12 Mädchen.

Jedes dieser Mädchen zählt die Anzahl an Jungen die direkt neben ihr Stehen.

Da es maximal 2 Personen neben Ihr sind kann sie 0, 1 oder 2 Jungen zählen. Ich habe die Zahlen die die Mädchen zählen direkt rechts neben die Aufstellung geschrieben.

Hallo

"Bestimme die zu erwartende Anzahl an Jungen, die neben einem Mädchen stehen"

ist nicht eindeutig, neben einem Mädchen können 10 Jungen stehen, wenn da nicht steht "direkt neben" Ich habe es als direkt neben interpretiert, und da stehen in deinem Beispiel 4 mal 2 Jungen direkt neben einem M und 2 mal nur 1 J direkt neben 1M. d,h, 1 ist das Minimum, kann also nich der EW sein genau wie bei 10 Jungen können 10 neben  einem M stehen oder nur einer und alle Möglichkeiten dazwischen, wieder ist 1 das Minimum,aber zuerst muss man wohl die Aufgabe richtig interpretieren, für mich ist 1 das Minimum, 10 das Maximum der EW muss dazwischen liegen.

lul.

Du verstehst anscheinend nicht wie ich es Interpretiert habe. Mir ist klar das es nicht eindeutig ist und ich habe auch gesagt das du es so interpretierst wie abakus. Ich es aber wohl so interpretiere wie Hikoba.

Wie die Frage tatsächlich gemeint ist kann nur der Urheber beantworten.

Hier also nochmal wie ich es interpretiere

Es stehen z.b. 10 Mädchen und 10 Jungen wahllos in einer Reihe.

Ein Mädchen betrachtet jetzt die Personen, die direkt neben Ihr stehen. Das kann eine Person sein, wenn sie am Rand steht oder auch 2 Personen, wenn sie mitten in der Reihe steht.

Das Mädchen fragt sich jetzt, wie viele Jungen im Mittel so neben Ihr stehen.

Da ein Mädchen ja entweder keinen, einen oder auch zwei Jungen direkt neben sich betrachten könnte, muss der Erwartungswert der Anzahl der Jungen neben diesem Mädchen irgendwo im Intervall von 0 bis 2 liegen.

Für n Mädchen und n Jungen haben Hikoba und ich es ausgezählt und sind im Mittel auf einen Jungen direkt neben einem Mädchen gekommen.

Über vollständige Induktion kann man zeigen, dass der Erwartungswert der Anzahl an Jungen, die direkt neben einem Mädchen stehen, immer 1 ist.

Achtung. Mir ist bewusst, wie ihr es interpretiert habt und weise explizit darauf hin dass ich es nicht so interpretiert habe.

Und warum erkläre ich das jetzt damit ihr versteht wie wir es interpretiert haben und das unsere Rechnung NICHT für den Fall eurer Interpretation anzuwenden ist.

Wer also die Rechnung von Hikoba verstehen will muss verstehen wie er es interpretiert hat. Wenn man es nicht versteht dann wird man die Rechnung selbstvertändlich auch nicht nachvollziehen können.

Ja, das ist häufiger das Problem bei Fragen von Hikoba (oder auch bei Fremdbeiträgen, an denen er beteiligt ist): Die Fragestellungen sind in der gestellten Form mehrdeutig und lassen mehrere Interpretationen zu.

Zur Klärung trägt er hier gar nichts bei. Man kann diesmal allerdings froh sein, dass hier (im Gegensatz zu anderen Threads) keinem von uns dann mangelndes Aufgabenverständnis vorgeworfen wird.

1 Antwort

0 Daumen

Könnt man vielleicht nachweisen, dass der Erwartungswert immer 1 ist? Mir kommt da irgendwie vollständige Induktion in den Sinn. Aber irgendwie scheint der Induktionsschritt nicht ganz so trivial zu sein.

Hast du dir das mit der vollständigen Induktion mal angesehen oder verstehst du davon zu wenig?

Der Trick ist aus gehend von einem Jungen und einem Mädchen für welche der Erwartungswert 1 ist zu zeigen. Das unter der Annahme es gilt für n Jungen und n Mädchen auch zu zeigen, dass es für n + 1 Jungen und n + 1 Mädchen gilt.

Ich habe jetzt zumindest eine Idee wie so ein Beweis gelingen könnte, habe mich aber selber noch nicht rangesetzt.

Avatar von 480 k 🚀

Ein anderes Problem?

Stell deine Frage

Willkommen bei der Mathelounge! Stell deine Frage einfach und kostenlos

x
Made by a lovely community